NBME 11 question

This forum made possible through the generous support of SDN members, donors, and sponsors. Thank you.

Master Deep

Full Member
15+ Year Member
Joined
May 16, 2008
Messages
154
Reaction score
1
Can anyone whos done NBME 11 explain how to figure out the serum protein electrophoresis question. 12 yr old boy admitted to hospital because of lethary, hip pain and fever. hes been admitted many times becaues of pneumonia. And then it gives the diff kinds of serum protein electrophoresis.
Thanks!

Members don't see this ad.
 
1. 25 man with sob and chest pain during exercising. Asthma, major depressive disorder. pulmonary finding? Crackles on the left base and apex, crackles on right base lung, decreasd breath sounds or increased wheezing?
There was a xray also that had a opacity on the right lower lung.


Can someone explain this one a little better for me. I remember I question similar to this in nbme 7, but the answer was A. the numbers where a little different, the p.value was .03 if I remember correctly. Same answer choices but the answer was not b.
can someone tell me if these are the right answers, thanks!

Two programs for the treatment of patients with newly detected hypercholestrolemia were tried in a community.Program A was used in one district of the community and program B was used in another.After four yrs 45% of the 2200 patients on program A and 49% of the 1900 patients on program B had been successfully treated for hypercholestrolemia.The difference between the success rates for the two programs was stastically significant pvalue is less than .01.Health officials however decided not to change to Program B in the 1st district because the magnitude of the difference was so small.Which of the following best explains their decision ?

A)They attributed the difference in success rate to chance alone
B)They distinguished between statistical significance and practical importance of the difference in success rates.
C)They felt the p value was too small to justify a decision in favor pf program B
D)They felt the samples were too small to justify a decision in favor of Program B

I think A, I saw some B's?

Increase in serum concentration in a diabetic after exercising, which enzyme is activated first?
ans: phosphorylase kinase?

3. 4yr girl burn on finger. has a blister. which cell is involved in re-epithelization of blister.
Basal layer kertinocytes, dermal dendritic cells , epitheial langerhans cells or macrophages

I seem to always get these skin repair questions wrong, can anyone direct me to a good source?

Thanks again :)
 
-- type 2 DM pt w/ chronic kidney failure, taking glipizide; Cr clearance is 20% of normal; lvls of Ca, PO4, PTH?

I thought (-) Ca, (-) PO4 ('cuz of hypovitamin D), (+) PTH... but this is wrong; is it (-) Ca, (+) PO4, (+) PTH ?
 
1.

Can someone explain this one a little better for me. I remember I question similar to this in nbme 7, but the answer was A. the numbers where a little different, the p.value was .03 if I remember correctly. Same answer choices but the answer was not b.

P-value is the probability that the observed difference (betw the 2 groups) will occur to chance alone. So a p-value of 0.03 would mean that 3% of the time, group A's mean will differ fr group B's mean due to pure chance. Anything less than p-value of 0.05 is usually considered "significant".

IIRC, the NBME 7 Q had a p value of 0.3 (?) .. anyway, the difference was not significant. 'Cuz 30% of the time, due to chance along, you'd see the difference between the 2 groups. Therefore the answer to that one was "they attributed the difference in success rate to chance alone".

1.
3. 4yr girl burn on finger. has a blister. which cell is involved in re-epithelization of blister.
Basal layer kertinocytes, dermal dendritic cells , epitheial langerhans cells or macrophages

I seem to always get these skin repair questions wrong, can anyone direct me to a good source?

Thanks again :)

I think I've seen this Q a couple times too on diff NBMEs. Maybe I under-thought this one, but I thought it was just trying to get at how the stratum basale layer has stem cells, to replace cells normally & in response to damage.
 
Members don't see this ad :)
55 y/o woman with CHF has SOB and chest pain, Xray shows bilateral pleural effusions.
Labs: pleural fluid has nucleated cell count of 500/mm
why?

-dec lymph drainage
-dec oncotic pressure
-dec serum proteins
-inc hydrostatic pressure
-inc vascular permeability (wrong) <-I chose this thinking WBCs leaked through during an inflammatory reaction

76 y/o woman taking prednisone for RA and HRP with estrogen+progest. normal Ca and VitD. x ray shows vertebral crush fractures. why?
-dec bone formation d/t dec cal absorbtion
-dec bone form d/t dec osteoblast differentiation
-inc bone resorb d/t dec cal absorbtion
-inc bone resorbtion d/t dec PTH
-inc bone resorb d/t estrogen receptor defect (wrong)

for immunicompromised pts, the PPD test is changed from 10mm to 5mm induration. whats the effect of incidence and prevalance for a positive PPD?
-i chose no change in either (wrong). can someone explain this one to me?
 
55 y/o woman with CHF has SOB and chest pain, Xray shows bilateral pleural effusions.
Labs: pleural fluid has nucleated cell count of 500/mm
why?

-dec lymph drainage
-dec oncotic pressure
-dec serum proteins
-inc hydrostatic pressure
-inc vascular permeability (wrong) <-I chose this thinking WBCs leaked through during an inflammatory reaction

increased hydrostatic pressure. Your main clue was the fact that the specific gravity of the fluid was 1.0018 or whatever, which is very low hence telling you it was a transudate NOT and exudate... permeability would be the answer if you had tons of proteins in there, specific gravity >1.02 roughly.

for immunicompromised pts, the PPD test is changed from 10mm to 5mm induration. whats the effect of incidence and prevalance for a positive PPD?
-i chose no change in either (wrong). can someone explain this one to me?

if you change the test and make it more sensitive this will allow more ppl to test positive, hence prevalence and incidence will both increase.
 
76 y/o woman taking prednisone for RA and HRP with estrogen+progest. normal Ca and VitD. x ray shows vertebral crush fractures. why?
-dec bone formation d/t dec cal absorbtion
-dec bone form d/t dec osteoblast differentiation
-inc bone resorb d/t dec cal absorbtion
-inc bone resorbtion d/t dec PTH
-inc bone resorb d/t estrogen receptor defect (wrong)

Lippincott's Pharm page 316. Prednisone causes decreased bone formation by blocking Calcium absoprtion in the gut, so her Ca + Vit D supplements weren't helping one bit ;)
 
Thanks buddy. I don't care what anyone says, that was a TOUGH assessment (dropped 12 points from NBME 12!)
 
Thanks buddy. I don't care what anyone says, that was a TOUGH assessment (dropped 12 points from NBME 12!)

I found them ALL super though! :s if you keep at it you'll be ok. If you feel like your scores are dropping take 1-2 days off questions and just study! It worked wonders for me. :luck:
 
Lippincott's Pharm page 316. Prednisone causes decreased bone formation by blocking Calcium absoprtion in the gut, so her Ca + Vit D supplements weren't helping one bit ;)

Ok, I've seen 2 diff answers for this in this thread. One is (-) Ca absorption, another is (-) osteoblasts to inhibit bone formation.

Can anyone confirm which one is actually correct?
 
Ok, I've seen 2 diff answers for this in this thread. One is (-) Ca absorption, another is (-) osteoblasts to inhibit bone formation.

Can anyone confirm which one is actually correct?

Coralis hit the nail on the head. It's s decrease in bone formation due to decreased calcium absorption.
 
15 mnth old white boy, with recurrent pneumonia and failure to thrive and fat greasy stools ,(cystic fibrosis), has a bro who has the same disease, bt her sisters are spared. has b/l rhonchi and cxr shows increased bronchovascular markings.. and there is this pedigree chart showing the affecteed child nd one of her cousin (girl) is also affected.
a- AD with variable expressivity
b- AD with variable penetrance
c- AR with high heterozygote freq
d- AR with low heterozygote freq( got this wrong)
e- X-linked rare gene
Any input would be appreciated... thanxin advance
 
Members don't see this ad :)
Input:
15 mnth old white boy, with recurrent pneumonia and failure to thrive and fat greasy stools ,(cystic fibrosis), has a bro who has the same disease, bt her sisters are spared. has b/l rhonchi and cxr shows increased bronchovascular markings.. and there is this pedigree chart showing the affecteed child nd one of her cousin (girl) is also affected.
a- AD with variable expressivity
b- AD with variable penetrance
c- AR with high heterozygote freq
d- AR with low heterozygote freq( got this wrong)
e- X-linked rare gene
Any input would be appreciated... thanxin advance
 
@ blonddoctor...can u pls explain...??

Ummm sorry, not a lot to explain. :) CF has a carrier frequency of 1 in 25 in some populations. Thus, its considered to have a high carrier (ie heterozygote) frequency.
Just don't overthink it. I feel like thats a common problem, sometimes there is more common sense then you would think to these questions. Take a minute and step back.
 
cystic fibrosis is AR.
in a normal situation, marrying into a family is expected to be normal (low frequency)
but in this case, the child has the disease that must means the person marrying into the family is a carrier(as oppose to normal). therefore, it seems that this disease has high frequency
 
cystic fibrosis is AR.
in a normal situation, marrying into a family is expected to be normal (low frequency)
but in this case, the child has the disease that must means the person marrying into the family is a carrier(as oppose to normal). therefore, it seems that this disease has high frequency

Why do you consider CF low freq?
 
Oh, whenever I see any pedigree problem, I just assume that the person (A) who is marrying into the family will be normal or less likely to be carrier for any given disease. In order for the child to be affected, person (A) must be carrier. Therefore, this disease must have high frequency. If it is low, you would expect person (A) would not be carrier. I also noticed that the affected cousin also has another marrying-into-the-family-carrier parent.

OR 1/25 would work, too. :)
 
25 yea old pilot goes 3 days wihtout food .. i answered "Glucagon activates glycerol utlilization as a source of carbon for gluconeogenesis by adipocytes" which is wrong bcoz as some one posted earlier gluconeo doesnt occur in adipocytes.
BUT glucagon activating hormone senstive lipase is also NOT TRUE. Source-> "LIPE is activated when the body needs to mobilize energy stores, and so responds positively to catecholamines, ACTH, and negatively to insulin. Previously, glucagon was thought to activate LIPE, however the removal of insulin's inhibitory effects ("cutting the brakes") is the source of activation. The lipolytic effect of glucagon in adipose tissue is minimal in humans.". <-- Wikipedia.
 
Lippincott's Pharm page 316. Prednisone causes decreased bone formation by blocking Calcium absoprtion in the gut, so her Ca + Vit D supplements weren't helping one bit ;)

I'm pretty sure the answer to this one was that corticosteroids inhibit osteoblast differentiation. I picked that and it didn't show up as wrong.
 
This question has been posted so many times, but I still don't get it.

A 52-year-old woman with type 2 diabetes mellitus has chronic renal failure. Creatinine clearance is 20% of normal.

What are the increase/derease of calcium/phosphorus/PTH?
Decrease in calcium/phosphate, Increase in PTH is wrong.

I thought in chronic renal failure, you have secondary hyperparathyroidism (decrease calcium, increase phosphate/PTH) or tertiary hyperparathyroidim (increase PTH/calcium). At least that's what is in First Aid.
Thanks for the help!
 
Last edited:
wouldnt it be C? Renal failure would cause decreased Ca and decreased phosphate clearance, due to renal failure. This would increase PTH.

I think...
 
couple questions here from NBME 11:

1. Drug promotes gastic emptying and releives N&V. I put ondansetron but it's wrong. Other choices: dronabinol, metoclompramide, misoprostol, scopolamine

2. Babinski sign is present bilaterally. These findings are most likely caused by a lesion in which of the following locations? I had put medial diencepholon but it was wrong
 
couple questions here from NBME 11:

1. Drug promotes gastic emptying and releives N&V. I put ondansetron but it's wrong. Other choices: dronabinol, metoclompramide, misoprostol, scopolamine

2. Babinski sign is present bilaterally. These findings are most likely caused by a lesion in which of the following locations? I had put medial diencepholon but it was wrong

You should look through before posting duplicate questions. (referencing to your first question) ....it's not that hard to look through 8 pages and do a control find

Question #2: answer is brainstem because babinski sign is affecting UMN (which is part of brainstem)
 
B is incorrect. I put that but got it wrong on the expanded feedback.:(

35 year old woman with leiomyomata uteri undergoes an experimental treatment that involves instillation of an embolizing agent directly to the vessels that feed the leiomyomata. During this procedure, in order to reach the involved vessels, which of the following is the most direct course of the catheter after entering the femoral artery?

A) Aorta-->gonadal artery-->uterine artery
B) External iliac artery-->internal iliac artery-->uterine artery
C)External iliac artery-->superior gluteal artery-->uterine artery
D)Internal iliac artery-->inferior vesical artery-->uterine artery
E)Internal iliac artery-->superior vesical artery-->uterine artery

The only other possibility I can think of is A, since ovarian artery and uterine artery may anastomose.
 
The only other possibility I can think of is A, since ovarian artery and uterine artery may anastomose.
Although the ovarian artery and uterine artery frequently anastomose, the most direct route is still External iliac artery-->internal iliac artery-->uterine artery
 
Another question:
Which is the first anatomical landmark visualized before reaching Larynx while doing Bronchoscopy:
1. Base of tongue
2. Epiglottis
3. False Vocal cord
4. True vocal cords
 
B is incorrect. I put that but got it wrong on the expanded feedback.:(
I did Form 11 today and got the same question. I marked b as my choice and it was not marked incorrect. So, it is the correct answer.
Are you sure yours was marked incorrect?
 
thanks!!

another ones...
randomized cohor study of drug X for MI found there was no decrese in mortlaity conpared to placebo , but on review of data, statisticlaly fewer death among drug X subjects in subgroup of nontransmural MI than in placebo group. a retrospective assessment of database for drug X supported the obserbation. which of the following is the most approrpiate next step.

a - cross sectional populationstudy of Drug X vs placebo after nontrasnmural MI
b - prospective randomized controlled study of drug x vs placebo after non transmural MI
c - tx of all pts with drug X after MI
d - tx of only pts with nontransmural MI with drug X
e - tx of only pts with transmural MI with drug X

i eliminated C and E.. and thought D bc the retrospective assessment supported the conclusion.. but thats wrong.. so it is b?
you guys are awesome. thanks for the help!.[/QUOTE]

So is this answer B? Anyone care to explain why?
 
So is this answer B? Anyone care to explain why?

It's kind of hard to explain without spending a lot of time talking about statistics, but in essence:

Obviously you have to choose between answer B and D. The reason that D is incorrect is because of the inherent error that comes with multiple comparisons, i.e. the more comparisons you run, the greater the chance is that you'll discover a difference which is solely attributable to chance and not to actual effect (type I error). The question stem implies that the researchers designed a study to assess the impact of X, failed to get the results they wanted and then did a post-hoc subgroup analysis which found a difference in a certain population. While what they did isn't entirely wrong per se, they can only really justify using those results to design a future study, not to change current therapeutic guidelines (especially in Step 1 world where everything needs a RCT in order to be valid).

Granted, I think this question is sort of a pain because this happens all the freaking time in published research, but it's a very important point that people sometimes don't realize -- especially when you have something like the big HTN / lipid drug trials where they have dozens of comparisons and may or may not actually correct for it.

Anyway, Wiki if you want to read more: http://en.wikipedia.org/wiki/Multiple_comparisons . I'm also definitely not a stats person, so someone might be able to do a better job explaining it, but that's a least a rudimentary explanation.
 
It's kind of hard to explain without spending a lot of time talking about statistics, but in essence:

Obviously you have to choose between answer B and D. The reason that D is incorrect is because of the inherent error that comes with multiple comparisons, i.e. the more comparisons you run, the greater the chance is that you'll discover a difference which is solely attributable to chance and not to actual effect (type I error). The question stem implies that the researchers designed a study to assess the impact of X, failed to get the results they wanted and then did a post-hoc subgroup analysis which found a difference in a certain population. While what they did isn't entirely wrong per se, they can only really justify using those results to design a future study, not to change current therapeutic guidelines (especially in Step 1 world where everything needs a RCT in order to be valid).

Granted, I think this question is sort of a pain because this happens all the freaking time in published research, but it's a very important point that people sometimes don't realize -- especially when you have something like the big HTN / lipid drug trials where they have dozens of comparisons and may or may not actually correct for it.

Anyway, Wiki if you want to read more: http://en.wikipedia.org/wiki/Multiple_comparisons . I'm also definitely not a stats person, so someone might be able to do a better job explaining it, but that's a least a rudimentary explanation.

Thanks, that makes sense.


I had it down to either A or B... and went with A, not really sure why.
 
Thanks, that makes sense.


I had it down to either A or B... and went with A, not really sure why.

Oh, my bad for the unnecessary long explanation -- it's not A because the study design is wrong. A cross-sectional study looks at prevalence data (of a disease, gene mutation, etc) at a certain point in time. It's not longitudinal at all so you can't track results from an intervention or anything like that.
 
For cirrhosis, you don't see much change in AST/ALT because there is very little liver tissue left.

Also to clarify the discussion on Selegiline - it is an MAO-B inhibitor at low doses, but at high doses (like the transdermal patch) - it is a non-selective MAO inhibitor.
 
If anyone can help with the following questions:

1) A 42 yo woman comes for a routine checkup, with 8yr history of HTN treated with a thiazide diuretic. She reports that she tires easily with exertion because of her weight but is otherwise asymptomatic. She drinks one to two glasses of wine weekly. There is no family history of liver disease. Her BMI is 29, BP 140/90. No other abnormalities except truncal obesity without striae. Serum studies show: BUN 19, Glucose 117, Creatinine 1, total cholesterol 227, HDL 32, Triglycerides 347, AST 87, ALT 85. Which of the following is most likely cause of increased liver enzyme activity.
a) alcoholic hepatitis
b) hemochromatosis
c) hepatitis A (WRONG)
d) nonalcoholic steatohepatitis
e) porphyria cutanea tarda

I don't think it could be (a) because will one glass of wine a week really cause increased ALT/AST? So is it (d)? Can someone explain?

2) A 38yo man comes with complaint of 3wk history of rectal pain and bleeding that's worse when he defecates. Similar symptoms 1 year ago. He is afebrile. Picture of anoscopy shows a mass in the anal canal. The patient has most likely which of the following?
a) anal fistula
b) colorectal carcinoma
c) condylomata acuminata (WRONG)
d) hemorrhoids
e) perianal abscess

I thought maybe he had anal cancer caused by HPV, hence choice C. Can someone explain?

3) Woman with PPD skin test. 25mm induration 48 hours later. Analysis of this patient's lesion is most likely to show predominance of:
a) B lymphocytes
b) Cytotoxic T lymphocytes (WRONG)
c) eosinophils
d) macrophages
e) mast cells
f) neutrophils

I knew PPDs were type IV hypersensitivity, so I went with the answer that's remotely related to T cells, but it was wrong. The next possible one that's remotely related to T cell mediated hypersensitivity will be macrophages, I guess?

4) 76yo woman with severe back pain for 2wks. No history of smoking. Taking prednisone for RA for 6 mons, and estrogen/progesterone for the past 15 yrs. Calcium and vit D intake are adequate. X-rays show vertebral fracture. Which of the following is the most likely cause?
a) decreased bone formation due to decreased Ca absorption
b) decreased bone formation due to inhibition of osteoblast differentiation
c) increased bone resorption due to decreased Ca absorption
d) increased bone resorption due to decreased PTH
e) increased bone resportion due to estrogen receptor defect (WRONG)

5) A 12yo boy with lethargy, hip pain, and fever. Previously hospitalized multiple times for pneumonia. CBCs are within normal limits, and HIV is negative. Blood cultures grow S. aureus. Serum electrophoresis is most likely to look like which pattern:
a) looks like a monoclonal gamma spike
b) the gamma hump and the beta2 hump are mushed together
c) increased gamma hump - widened, not spiked (WRONG)
d) no gamma hump
e) normal pattern
I somehow missed the clues for the fact that this boy has multiple myeloma (so a would be correct). I just wanted someone to confirm.
 
If anyone can help with the following questions:

1) A 42 yo woman comes for a routine checkup, with 8yr history of HTN treated with a thiazide diuretic. She reports that she tires easily with exertion because of her weight but is otherwise asymptomatic. She drinks one to two glasses of wine weekly. There is no family history of liver disease. Her BMI is 29, BP 140/90. No other abnormalities except truncal obesity without striae. Serum studies show: BUN 19, Glucose 117, Creatinine 1, total cholesterol 227, HDL 32, Triglycerides 347, AST 87, ALT 85. Which of the following is most likely cause of increased liver enzyme activity.
a) alcoholic hepatitis
b) hemochromatosis
c) hepatitis A (WRONG)
d) nonalcoholic steatohepatitis
e) porphyria cutanea tarda

I don't think it could be (a) because will one glass of wine a week really cause increased ALT/AST? So is it (d)? Can someone explain?

When you've ruled everything else out, go with NASH. She's fat -> fatty liver.

2) A 38yo man comes with complaint of 3wk history of rectal pain and bleeding that’s worse when he defecates. Similar symptoms 1 year ago. He is afebrile. Picture of anoscopy shows a mass in the anal canal. The patient has most likely which of the following?
a) anal fistula
b) colorectal carcinoma
c) condylomata acuminata (WRONG)
d) hemorrhoids
e) perianal abscess

I thought maybe he had anal cancer caused by HPV, hence choice C. Can someone explain?

Hemorrhoids. These can come and go, while warts don't. Plus, the picture looked like hemorrhoids (mucosal folds rather than verrucous warts). Fistula would be a possibility with Crohns, CRC wouldn't come and go like this, afebrile so no abscess.

3) Woman with PPD skin test. 25mm induration 48 hours later. Analysis of this patient’s lesion is most likely to show predominance of:
a) B lymphocytes
b) Cytotoxic T lymphocytes (WRONG)
c) eosinophils
d) macrophages
e) mast cells
f) neutrophils

Memory T Cells (not cytotoxic t cells) recruit macrophages in the PPD test.

I knew PPDs were type IV hypersensitivity, so I went with the answer that’s remotely related to T cells, but it was wrong. The next possible one that’s remotely related to T cell mediated hypersensitivity will be macrophages, I guess?

4) 76yo woman with severe back pain for 2wks. No history of smoking. Taking prednisone for RA for 6 mons, and estrogen/progesterone for the past 15 yrs. Calcium and vit D intake are adequate. X-rays show vertebral fracture. Which of the following is the most likely cause?
a) decreased bone formation due to decreased Ca absorption
b) decreased bone formation due to inhibition of osteoblast differentiation
c) increased bone resorption due to decreased Ca absorption
d) increased bone resorption due to decreased PTH
e) increased bone resportion due to estrogen receptor defect (WRONG)

Prednisone is screwing with the osteoblasts/osteoclasts. Both increased bone resorption and decreased bone formation through transcriptional control.

5) A 12yo boy with lethargy, hip pain, and fever. Previously hospitalized multiple times for pneumonia. CBCs are within normal limits, and HIV is negative. Blood cultures grow S. aureus. Serum electrophoresis is most likely to look like which pattern:
a) looks like a monoclonal gamma spike
b) the gamma hump and the beta2 hump are mushed together
c) increased gamma hump - widened, not spiked (WRONG)
d) no gamma hump
e) normal pattern
I somehow missed the clues for the fact that this boy has multiple myeloma (so a would be correct). I just wanted someone to confirm.

Supposed to be agammaglobulinemia (recurrent pneumonias). Not multiple myeloma. Thought it was kind of a ****ty stem for agammaglobulinemia.

Derp.
 
Thanks, man. One more question - so the guy with the hemorrhoids...Since you can see them with anoscopy, I assume they're INTERNAL hemorrhoids, right? But then, I thought internal hemorrhoids weren't painful...and this guys says "painful on defecation." Any thoughts on that? (I agree that none of the other choices make sense though...but because of the pain, I ruled out internal hemorrhoids and went for possible anal cancer. Idk.)
 
Did anyone ever post a response to the CT question about the tumor invading the pericardium?

A 56 year old woman has recently diagnosed carcinoma of the breast. An x-ray showed tumor next to the right side of the heart. An enhanced CT showed the tumor invading the pericardium. Which is most likely involved?
-Coronary sinus (WRONG)
-Greater splanchnic vein
-Right phrenic nerve
-Right vagus nerve
-Thoracic duct

Thanks guys. I agree - this test was relatively difficult. I take step 1 in 10 days and this kicked me into high(er) gear, so at least something good came of it.

EDIT: sorry, saw the correct answer was posted, deleted, but quoted in someone else's answer on another page. It's "phrenic nerve," I guess, for those interested.
 
Last edited:
Thanks, man. One more question - so the guy with the hemorrhoids...Since you can see them with anoscopy, I assume they're INTERNAL hemorrhoids, right? But then, I thought internal hemorrhoids weren't painful...and this guys says "painful on defecation." Any thoughts on that? (I agree that none of the other choices make sense though...but because of the pain, I ruled out internal hemorrhoids and went for possible anal cancer. Idk.)

I think that was hemorrhoid, it is definitely not anal cancer.
 
An 80 yo woman is brought to the ER after she was found wandering the street confused. She has no medical conditions and medications. She states she wants to go home. Oriented to person, not time and place. Only able to provide home address.

A. Try to contact family
B. Arrange for transport to her home
C. Arrange for transfer to nursing facility
D. Administer sedative
E. Admit pt to psych center
.

Remember determining one's competency is up to a judge. THe doctor cannot make the decision herself/himself. THerefore, anything that robs the patient of autonomy at this point is unethical.

B) is wrong because this is abandoning the patient.
C) is wrong because it robs autonomy
D) is wrong because it may be construed as maleficence
E) is wrong because it robs autonomy

Wouldn't contacting her family be a violation of HIPAA, and the fact that she only had a home address (no phone number) means you can't really contact her family? I mean I guess the keyword here was "try" but then why would they even put that sentence in there.. so I eliminated it and then I just picked transfer to nursing facility b/c she had confusion and was not oriented to time and place. Also, b/c she was confused and not oriented, I was thinking of what I would do if it were a drunk patient, and if the patient was drunk and wanted to go home, I would rob them of their autonomy and keep them in the hospital. I got the question wrong though so obviously my reasoning was wrong.. can anybody explain?

also does anybody know of good resources for behavioral science? any thoughts on BRS or HY behavioral science? people say it's a short read but honestly it looks pretty long to me.. but on my score report behavioral science was my worst section so maybe it might be worth it.
 
Imagine if your mother had Alzheimer's. Now imagine you were at work and left her home that day, assuming that she would not get in to any trouble (you take the car keys and lock the doors). She finds a way out of the house, and starts walking down the street. A doctor finds her. You really think the correct response is to take her and commit her to a nursing facility, rather than just giving you a call?

And contacting the family is not a violation of HIPAA because she can't make decisions for herself. It's like if someone is in a coma, the family makes decisions.
 
thanks! i guess sometimes common sense is the best way.. but i definitely need to brush up on when HIPAA applies and when it doesn't
 
Can someone explain the question that tested 10,000 people and showed negative vs. positive tests in people with and without the disease on a graph and asked where you should set the cut-off for the test? They talked about how it's dangerous to miss people with the disease and not a big deal to treat the non-diseased. Anyhow, I must be misunderstanding the curves or something.

Edit: Oh wow, sheesh, just looked at it again. It's like an optical illusion. Saw that someone else made the same mistake. What a silly question to miss.
 
Last edited:
Top